Get Answers to all your Questions

header-bg qa

In Figure 1, BL and CM are medians of a  \triangle ABC right-angled at A. Prove that 4\left ( BL^{2}+CM^{2} \right )= 5\, BC^{2}

 

 
 
 
 
 

Answers (1)

Given in \triangle ABC\rightarrow \angle A= 90^{\circ};B\angle and CM are medians
To Prove \rightarrow 4\left ( BL^{2}+CM^{2} \right )= 5BC^{2}

AL= \left ( \frac{AC}{2} \right )---(i)\left ( \therefore AL= LC \right )
AM= \left ( \frac{AB}{2} \right )---(ii)\left ( \therefore AM= BM \right )
Now in \triangle ABC,\triangle BAL\, and\; \triangle MBC
BC^{2}= AB^{2}+AC^{2}---(iii) (using pythagarous in \triangle ABC)
CM^{2}= AC^{2}+BM^{2}---(iv) (uisng pythagarous in \triangle MAC)
BL^{2}= AB^{2}+AL^{2}---(v) (using pythagarous in \triangle BAL)
Adding equation (iv) and (v)
CM^{2}+BL^{2}= AB^{2}+AC^{2}+AL^{2}+AM^{2}
CM^{2}+BL^{2}= BC^{2}+AL^{2}+AM^{2} (using (iii))
CM^{2}+BL^{2}= BC^{2}+\frac{AC^{2}}{4}+\frac{AB^{2}}{4} (using (i) and (ii))
CM^{2}+BL^{2}= \frac{4BC^{2}+AC^{2}+AB^{2}}{4}
\Rightarrow 4\left ( CM^{2}+BL^{2} \right )= 5BC^{2} (using (iii))
Hence proved

Posted by

Safeer PP

View full answer